IS曲线和LM曲线的移动如何影响收入和利息率

一个人的生日2022-10-04 11:39:541条回答

已提交,审核后显示!提交回复

共1条回复
艺术商人 共回答了19个问题 | 采纳率94.7%
IS,LM曲线移动时,会影响收入和利息同时变动.
当LM不变,IS向右上方移动时,收入增加,利率上升.
当LM不变,IS向 左下方移动时,收入减少,利率下降.
当IS不变,LM向右下方移动时,收入增加,利率下降.
当IS不变,LM 向左上方移动时,收入减少,利率上升.
1年前

相关推荐

财政政策为什么在LM曲线越平坦,IS曲线越陡峭的时候作用大?
心静如水50501年前1
影子里的圆舞曲 共回答了19个问题 | 采纳率73.7%
其实你只要这样想,LM曲线越平坦,就越接近凯恩斯陷阱区域,财政政策最为有效,IS曲线则刚好相反.
具体分析的话,IS曲线是r=(a+e)/d-(1-b)*y/d,当IS曲线越陡峭时,就说明斜率越大.此时d也越,说明投资对利率变化的反应大,此时增加政府支出,利率上升,引起的投资减少较多,从而挤出效应大,财政效果小.
同理,LM曲线是r=k*Y/h-m/h,LM曲线越平坦说明斜率越小,h越大,L=ky-hr,h则越大,利率变动引起hr变动大,而L即货币供给量不会变,必然使得收入也相应增加.财政效果明显.
出口x增加,is曲线的移动等于x的变化量除以乘数,为什么是除呀?应该什么时候除,什么时候乘呀
CAY插1年前1
dryriver 共回答了12个问题 | 采纳率100%
你们是哪个学校的啊?为什么把宏观经济学教成数学一般.宏观经济学重要的是思想,而不是数学.希克斯把凯恩斯的思想数学化了,他俩哪个更伟大?当然还是人家凯恩斯伟大.数学只是工具而已.
你问的这个也是数学问题.你画一条IS线,你X增加100,整条线上的所有点都得向上移动100,而且新线是原线的平行线.
现在人家问你IS曲线的移动量,平行线的移动量当然是指平行线的距离.算距离时,当然得拿这100除以这个线的斜率.IS线的斜率不就是乘数么?
什么时候乘以,什么时候除以,这个没有规律啊.再说你知道这规律,除了考试,还有什么用处啊
宏观经济学中IS曲线的斜率 为什么是 dr/dy? 收入y不是因变量吗?为什么推导的时候却是对r 求导?
天地方园1年前1
小乔9uu 共回答了15个问题 | 采纳率80%
首先,IS曲线上的r和y并非因果关系,只是说明,当产品市场均衡时r与y之间存在这种组合关系;其次,从IS坐标图看,纵坐标为r,横坐标为y,所以IS曲线的斜率 是 dr/dy(换句话说,如果你把纵坐标设为y,横坐标设为r,那么IS曲线的斜率就变为 dy/dr.).
第三,r和y并非因果关系,那么在推导的时候,采用dr/dy,还是dy/dr,就看哪种表达更方便.
求宏观经济学计算设IS曲线为y=5400-4000r,LM曲线为y=4800+2000r(1)求均衡产出和均衡利率(2)
求宏观经济学计算
设IS曲线为y=5400-4000r,LM曲线为y=4800+2000r
(1)求均衡产出和均衡利率
(2)若充分就业的总产出为5500,在货币政策保持不变的条件下,政府应该增加多少购买支出才能实现这一目标,实现经济将会怎么变动?
(3)若央行为维持利率不变而增加货币供给量,政府为实现充分就业而需要增加多少政府支出?货币供给需要增加多少?
浪漫七号1年前1
xie_Benson 共回答了15个问题 | 采纳率86.7%
1.均衡利率在IS曲线与LM相交时出现,联立两个方程求出利率r=0.1=10%
此时的产出为均衡产出y=5000
2.充分就业总产出为5500时,货币政策不变,看LM曲线,求出r=0.35
代回IS曲线,此时IS中:y=5400-4000*0.35=4000
所以政府还应增加(5500-4000)=1500的支出.
推导IS曲线,产品市场均衡 利率和储蓄为什么成反方向变动?利率下降或上升,储蓄应该减少或增加啊
137373236311年前1
xiaoyan_369 共回答了14个问题 | 采纳率71.4%
产品市场均衡,意思就是i=s,投资=储蓄.利率上升,投资减少,所以储蓄也减少,利率下降相反.关键是i=s,说明利率上升,投资减少,总收入下降,总收入下降导致储蓄下降.
问题中说的利率下降储蓄下降也是可能的,那条件就不是产品市场均衡,而是利率下降,总收入不变,这种情况则储蓄会下降,但是这种情况没有达到均衡
(a)的IS曲线为y=750-25r; 同理可解得(b)的IS曲线为y=750-50r
(a)的IS曲线为y=750-25r; 同理可解得(b)的IS曲线为y=750-50r
(a)的IS曲线为y=750-25r;(b)的IS曲线为y=750-50r
比较(a)和(b),说明投资对利率更敏感时,IS曲线的斜率发生什么变化
答案是((b)的投资函数中的投资对利率更敏感,表现在IS曲线上就是IS曲线斜率的绝对值变小,即IS曲线更平坦一些)
表现在IS上不是斜率绝对值变大吗50>25啊?不是更陡峭吗?
未知数32341年前1
kof_ljx 共回答了22个问题 | 采纳率95.5%
我觉得你的判断是正确的.投资对利率更敏感时,答案的确是b,但b的斜率的绝对值更大,曲线更陡峭.在r变化相同的情况下,b比a对r的变化更敏感.
为什么IS曲线的斜率是负值,LM曲线的斜率为正?
为什么IS曲线的斜率是负值,LM曲线的斜率为正?
为什么IS曲线的斜率是负值,LM曲线的斜率为正
千秋大业1年前1
Evania 共回答了26个问题 | 采纳率96.2%
IS曲线是反映当产品市场处于均衡状态时利率与国民收入之间的关系.所谓均衡状态就是I=S,即投资等于储蓄.因为投资是利率的减函数,储蓄是国民收入的增函数,当投资等于储蓄时,可导出国民收入是利率的减函数,表示这个关系的曲线就是IS曲线,那么它的斜率一定是负值.它表示 :当产品市场处于均衡状态时,国民收入随利率的增加而减少. 这个也可以这样解释:当利率上升的时候,投资会减少,投资的减少会通过乘数作用使国民收入减少.
LM曲线是反映当货币市场处于均衡状态时,利率与国民收入的关系.所谓均衡状态就是L=M
L是货币需求函数,是利率和国民收入的函数,即随利率的升高而减少,随国民收入的增加而增加.M是定值.所以当L=M时,可推出利率与国民收入是正相关的关系.即LM曲线的斜率为正.
关于IS-LM曲线的.什么情况下IS曲线会向左或向右移动.同理LM也是.
mpd151年前1
我向前 共回答了22个问题 | 采纳率90.9%
我们以一个最简单的四部门IS曲线和一个最简单的LM曲线方程为例:
IS:r=(α+e+G-βT+βTr+NX)/d-(1-β)Y/d,r是利率,α是自主性消费支出,e是自主性投资支出,G是政府财政支出,T是固定税收,Tr是政府转移支付,NX是净出口,β是边际消费倾向,d是投资的利率弹性,Y表示收入.
LM:r=kY/h-m/h,k是货币需求的收入弹性,h是货币需求的利率弹性,m是实际货币供给.
显然,在以利率r为纵坐标、收入Y为横坐标的IS-LM模型中,政府财政支出G增加,IS向右移动,反之则向左;政府税收T增加,IS向左移动,反之则向右;净出口NX(出口X-进口M)增加,IS向右移动,反之则向左;居民自主性消费α增加,IS向右移动,反之则向左;政府转移支付Tr增加,IS向右移动,反之则向左.
货币供应量m增加,LM向右移动,反之则向左.
总之,扩张型财政政策使IS向右移动,紧缩性财政政策使IS向左移动;扩张型货币政策使LM向右移动,紧缩性货币政策使LM向左移动.
宏观经济学问题,高手进IS曲线倾斜表示( )之间的关系。A.利率与利率影响下的均衡收入 B.收入与收入影响下的均衡利率C
宏观经济学问题,高手进
IS曲线倾斜表示( )之间的关系。
A.利率与利率影响下的均衡收入 B.收入与收入影响下的均衡利率
C.商品与货币市场均衡的收入与利率D.政府购买与收入
这题为什么选B,IS曲线下不应该是利率影响投资储蓄从而影响收入么?
cghkhjljhv71年前1
勇者的心 共回答了24个问题 | 采纳率87.5%
IS曲线表示的是收入(y)与利率(r)之间的关系,其倾斜方向就表示二者之间负相关关系
宏观经济学高鸿业鸿观经济学,第四版,第十七章第一节,有这样一句话,“价格水平的变化,对IS曲线的位置没有影响.这是因为,
宏观经济学
高鸿业鸿观经济学,第四版,第十七章第一节,有这样一句话,“价格水平的变化,对IS曲线的位置没有影响.这是因为,决定IS曲线的变量被假定是实际量,而不是随价格变化而变动的名义量.”我非常的不明白,名义货币量不应该是票面值的货币量吗?那么应该是实际量由名义量和价格水平决定才对啊,为什么书上说随价格变化而变动的是名义量呢?到底随价格变化而变动的是名义量还是实际量?
牛牛19791年前1
rr无奈的rr 共回答了16个问题 | 采纳率87.5%
IS曲线的变量是假定的实际量,实际量是由名义量和价格水平决定的.当价格变化时,若是实际量不变(这是默认的),则名义量会随价格变化而变动,但由实际量决定的IS曲线位置没影响.
个人认为是你对语意没理解,可能是国文理解上的问题,或者是脑袋一时转不过来.
宏观经济学 关于IS曲线的一个方程.怎么解
当兵的人68681年前1
专封斑猪ID 共回答了20个问题 | 采纳率80%
你解答的不等式中下一个方程写错了,变为r=(ky-m)/h,然后带入上个方程中求解即可
你的草稿纸上面写着r=y是不对的
请问宏观经济学中的IS曲线在现实经济中的意义是什么?觉得IS曲线很抽象,没什么实际意义.
avbt1年前1
Cissy_1025 共回答了15个问题 | 采纳率86.7%
好多刚接触宏观经济学的人都会有类似的想法,IS曲线,I代表的是Investment也就是投资,S代表的是Saving也就是储蓄.IS曲线代表的就是产品市场均衡曲线,也就是满足I=S这个均衡条件时各种利率r和收入Y的点的集合所构成的曲线.IS曲线的推导,任何一本宏观经济学教科书上都有,我简单阐述一下,随着利率的下降,投资会增加,随着投资的增加,国民收入会提高,因此IS曲线斜率为负.当然上面说的是只有两个部门的情况,我们也可以把IS曲线扩展成三部门,四部门的情况.分别是S+T=I+G和S+T+M=I+G+X.IS曲线上的点就是满足均衡条件的利率和收入的组合.IS曲线下方的点意味着I>S,IS曲线上方的点意味着I
IS曲线为什么向右下方倾斜?
99009104251年前1
njxun 共回答了26个问题 | 采纳率84.6%
在产品市场达到均衡时,收入和利率的各种组合的点的轨迹.在两部门经济中,IS曲线的数学表达式为I(R)=S(Y) ,它的斜率为负,这表明IS曲线一般是一条向右下方倾斜的曲线.一般来说,在产品市场上,位于IS曲线右方的收入和利率的组合,都是投资小于储蓄的非均衡组合;位于IS曲线左方的收入和利率的组合,都是投资大于储蓄的非均衡组合,只有位于IS曲线上的收入和利率的组合,才是投资等于储蓄的均衡组合
IS曲线表示满足什么关系
KANG126QIAN1年前3
spring505 共回答了19个问题 | 采纳率100%
IS曲线的含义:IS曲线是描述产品市场均衡时,利率与国民收入之间关系的曲线,由于在两部门经济中产品市场均衡时I=S,因此该曲线被称为IS曲线.
IS曲线中的收入指国民收入还是可支配收入?
克敏1年前1
洛瑟佛维斯 共回答了18个问题 | 采纳率100%
y指国民收入
宏观经济学IS曲线问题3、假定某经济中消费函数为C=0.8(1-t)Y,税率t=0.25,投资函数为I=900-50r,
宏观经济学IS曲线问题
3、假定某经济中消费函数为C=0.8(1-t)Y,税率t=0.25,投资函数为I=900-50r,政府购买G=800,货币需求为L=0.25Y+200-62.5r,实际货币供给为M/P=700,试求:(1)IS曲线(2)LM曲线(3)两个市场同时均衡时的利率和收入。
答案1. Y=C+I+G=0.8(1-t)Y+900-50r+800=0.8Y-0.8tY-50r+1700 将t=0.25带入并化简得到:
IS Y=-125r+4250
2. L=M/P 得到LM Y=250r+2000
3. 同时均衡解出: r=6 Y=3500
第一问求IS曲线为什么不能用
Y=(α+e-dr+g-βt)/1-β 这一公式
teczt1年前1
独臂剑客 共回答了23个问题 | 采纳率91.3%
因为你给的哪个公式是定量税,可是这个题是比例税呀。所以Y=(α+e-dr+g)/(1-β(1-t)),自己推一推公式就好了
IS曲线斜率为什么是负值(经济原因)
gloriaxyz1年前1
ctgyang 共回答了20个问题 | 采纳率85%
is 是investment saving curve
其中投资和是利率的负相关,储蓄是收入的正相关
is是所有储蓄和投资相等的均衡点
那么如果利率很高,投资就会很少,达到均衡时候的储蓄也会很少,自然对应的收入就很少了
,所有高利率和低收入是在同一点,相反,低利率和高收入对应的也是低利率带来的高投资等于高收入带来的高储蓄的均衡.斜率自然也就是负的(利率和收入在曲线上为负相关)
IS曲线的含义是什么
g1gr2ob1年前1
ADSFZZ 共回答了17个问题 | 采纳率100%
IS曲线概述  IS曲线的含义:IS曲线是描述产品市场均衡时,利率与国民收入之间关系的曲线,由于在两部门经济中产品市场均衡时I=S,因此该曲线被称为IS曲线.   在产品市场达到均衡时,收入和利率的各种组合的点的轨迹....
消费增加为什么能让IS曲线右移?
cantonadam1年前1
bright_tt 共回答了27个问题 | 采纳率96.3%
在IS曲线里,消费C是函数中常数项之一.在两部门经济中
S=Y-C,
I=e-dr,
所以IS曲线的函数形式为
r=(C+e)/d-Y/d
由此不难看出消费C与IS曲线的关系.
IS曲线为什么向右下方倾斜?
dula131年前1
蜗牛真幸福 共回答了17个问题 | 采纳率94.1%
在产品市场达到均衡时,收入和利率的各种组合的点的轨迹.在两部门经济中,IS曲线的数学表达式为I(R)=S(Y) ,它的斜率为负,这表明IS曲线一般是一条向右下方倾斜的曲线.一般来说,在产品市场上,位于IS曲线右方的收入和利率的组合,都是投资小于储蓄的非均衡组合;位于IS曲线左方的收入和利率的组合,都是投资大于储蓄的非均衡组合,只有位于IS曲线上的收入和利率的组合,才是投资等于储蓄的均衡组合
设两部门经济的消费方程为C=500+0.5Y,投资函数为I=1250-250r.求该经济的IS曲线方程.
设两部门经济的消费方程为C=500+0.5Y,投资函数为I=1250-250r.求该经济的IS曲线方程.
在上述经济体中,若货币需求为L=1000+0.5Y-250r,货币供给为M=1250,求该经济体的LM方程.
junhill1年前1
舒葶 共回答了20个问题 | 采纳率100%
C=500+0.5Y
所以α=500,β=0.5
I=1250-250r
所以e=1250,d=250
IS=α+e/d+(1-β/d)Y=7-0.02Y
(2)L=1000+0.5Y-250r
∴K=0.5 ,h=250
m=1250/1ooo
LM=(K/h)Y-m/h=0.02Y-0.OO5
两部门经济IS曲线 投资增加 is曲线移动方向 为什么是向右上方移动?
听你说书1年前1
xiao_001 共回答了14个问题 | 采纳率71.4%
在利率不变的情况下,投资增加会使产出增加,IS曲线自然是向右移
急!霍尔实验思考题:根据所得的VH~及Im及VH~Is曲线,能否设计一种用霍尔元件制成的电流表.
28595031年前1
梦醒时分5566 共回答了16个问题 | 采纳率87.5%
能设计,由V-I图像可知当外加磁场B和流径霍尔原件的电流I的其中一个值衡定时,霍尔电压与另外一值成正比关系,因此可以选择电阻率较低的霍尔原件设计电流表,只要得出霍尔电压,根据V-I关系即可读出电流I的值.
位于IS曲线以外的点是如何表示产品市场非均衡状态的?利用IS曲线说明产品市场的自动均衡机制.
永远懂你1年前1
南风劲 共回答了17个问题 | 采纳率82.4%
参考答案x09红泥地又叫血地,一铲下去泥中带血,谁也无法解释这种地象是怎么形成的,但是所有的风水流派中,对于血地埋尸的说法都是惊人的一致,那就是适宜深埋,葬于此地刹气极重,后代必然极其显贵云深但是是亲戚死绝,说不定能当皇上,但是家里人全部都会给克死.
画图说明在两部门模型中,IS曲线的推导,并画出在政府支出增加的情况下,IS曲线是怎样移动的.
顺手一巴掌1年前1
vikingzp 共回答了18个问题 | 采纳率94.4%
IS曲线:I=S
把投资线和储蓄线画出後,相交点连接就是IS线
政府支出增加,使投资增加,相同收入,投资增加,代表IS线向上/向右移动
is曲线满足以下哪一种关系?
梦想之婕1年前1
浓情巧克力 共回答了17个问题 | 采纳率100%
产品市场均衡的关系
总支出=总收入 的关系
IS曲线中r表示什么是资本边际效率还是市场利率
yemao2501年前1
ii者的天灾石 共回答了20个问题 | 采纳率75%
市场利率
利率升高 会抑制投资和消费?为什么 is曲线说的是 产品市场均衡 二部门条件下 是i=s ,我知道 利率升高 会抑制投资
利率升高 会抑制投资和消费?为什么 is曲线说的是 产品市场均衡 二部门条件下 是i=s ,我知道 利率升高 会抑制投资和消费?为什么
is曲线说的是 产品市场均衡 二部门条件下 是i=s ,我知道i=e-dr 所以 投资减少 那消费的话是为什么 (其实 没怎么学好is曲线 )如果i=s的话 s=y-(a+by)那么s 是不是也减少了? 那么c我觉得是增加啊 还是r升高了 人们边际消费倾向减少了 这样的话 那么s是增多了么? 那不是 投资不等于储蓄了么……实在不懂 求解答
我不清楚为什么产品市场 供给是 c+s 需求是c+i 诶 那节课没听……求大神
tianyimore1年前1
keren0128 共回答了15个问题 | 采纳率93.3%
利率升高,人更倾向于去储蓄,贷款也会减少,手中的货币量减少当然更倾向于减少投资和消费.这不一定需要从数学模型中推理出来,从常理来就可以推断出来.
设消费函数C=100+0.75Y,投资函数i=20-2r,货币需求L=0.2Y-0.5r,货币供给M=50.求IS曲线和
设消费函数C=100+0.75Y,投资函数i=20-2r,货币需求L=0.2Y-0.5r,货币供给M=50.求IS曲线和LM曲线的函数表达
宏观经济 第四版 高鸿业主编
ldy07121年前1
flylongsky 共回答了14个问题 | 采纳率100%
IS:Y=C+I,Y=100+0.75Y+20-2r,Y=480-8r;
LM:L=M,0.2Y-0.5r=50,Y=250+2.5
is曲线移动储蓄减少,则投资增加.但是既然is的条件是i=s,那不就是说s减少了,投资也应该减少吗,这个怎么理解,现在我
is曲线移动
储蓄减少,则投资增加.但是既然is的条件是i=s,那不就是说s减少了,投资也应该减少吗,这个怎么理解,现在我还没有分啊,以后有了可以补上!
天恩和海霞1年前1
掌中舞 共回答了22个问题 | 采纳率100%
IS曲线总结了利率和收入水平之间的关系.实质上IS曲线结合了投资函数表示的利率与投资之间的作用和凯恩斯交叉图表示的收入和投资之间的相互作用.
所以IS的条件不能简单的说成I=S,储蓄的减少可能是因为利率下降了,利率的下降必然会使投资增加.
你可以看看IS曲线的推导过程.
如果IS曲线为水平时,使用扩张性财政政策,则IS曲线如何移动
jsl19831年前0
共回答了个问题 | 采纳率
如果储蓄减少导致IS曲线向什么方向移动,这是为什么,希望有图解,
HRSEM1年前1
gelinqing 共回答了18个问题 | 采纳率94.4%
IS曲线向右上方移动
IS曲线的移动 :投资增加或者储蓄减少,IS曲线向右上方移动;投资减少或者储蓄增加,IS曲线向左下方移动.
①投资函数的移动,如果某种原因使自主投资e增加,投资曲线向右上方移动,IS向右上方移动.反之e减少,IS左下方移动.
②储蓄函数移动,人们储蓄愿望增强,每一收入水平上人们增加了储蓄,储蓄曲线向左上方移动,IS左下方移动.反之IS右上方移动.
3.本国货币贬值,有利于进口增加,不利于出口.正确 错误 4.IS曲线是一条向右下方倾斜的曲线 正确 错误
3.本国货币贬值,有利于进口增加,不利于出口.正确 错误 4.is曲线是一条向右下方倾斜的曲线 正确 错误
5.凯恩斯的主义货币***和货币主义的货币***作用的机理是一样的.
正确 错误
单选题
1.is—lm模型研究的是
a.在利息率与投资不变的情况下,总需求对均衡的国民收入的决定
b.在利息率与投资变动的情况下,总需求对均衡的国民收入的决定
c.在利息率与投资不变的情况下,总需求对均衡的国民收入的决定.
d.在总供给不变的情况下总需求对均衡的国民收入的决定
2.在历史发展的各个不同时期,实行过多种汇率***,大体上可以分为
a.直接汇率b间接汇率c固定汇率制与浮动汇率制d以上都对
3.假定货币供给量不变,货币的交易需求和预防需求的增加将导致货币的投机需求
a不变b增加c减少d不能肯定
4.引起摩擦性***的原因
a工资能升不能降的刚性b总需求不足c经济中劳动力的正常流动d技术进步
5.属于内在稳定器的******工具是
a***福利的支出b***公***程支出c***购买d货币供给
6.is曲线向左下方移动的条件是
a自发总需求增加b自发总需求减少c价格水平下降d价格水平上升
7.属于内在稳定器的******工具是
a***福利的支出b***公***程支出c***购买d货币供给
8.影响各国之间经济开放程度较小的因素是
a边际消费倾向b边际进口倾向c***的大小d开放程度
9.根据需求管理的原理,抑制总需求的条件是
a总需求大于总供给b总需求等于总供给c总需求小于总供给d经济中出现严重衰退
10.根据菲利普斯曲线,降低通货膨胀率的办法是根据菲利普斯曲线,降低通货膨胀率的办法是
a减少货币供给量b降低***率c提高***率d增加工资
多选题
1.实行赤字***
a在短期内可以刺激经济增长
b在长期内可以刺激经济增长
c在经济萧条时使经济走出衰退
d可以使经济持久繁荣
e对经济没有影响
2.按照价格上涨幅度加以区分,通货膨胀包括
a温和的通货膨胀b奔腾的通货膨胀c平衡式通货膨胀d非平衡式通货膨胀e***的通货膨胀
3.影响进口的直接因素是
a初始进口b边际进口倾向 c国民收入d边际消费倾向e消费者剩余
4.按***产生的原因,可将***分为
a摩擦性***b结构性***c周期性***d自愿性***e季节性***
5.lm曲线
a描述产品市场达到均衡时,国民收入与价格之间的关系
b描述货币市场处于均衡的,收入与均衡利率的不同组合出来的一条曲线
c描述产品市场处于均衡的,收入与均衡利率的不同组合出来的一条曲线
d'lm曲线向右上方倾斜e'lm曲线向右下方倾斜
6.属于内在稳定器的项目是
a***购买b税收c***转移支付d***公***程支付e以上说法均正确
7.影响is曲线移动的因素有
a如果投资增加,则is曲线向右上方移动
b***购买增加,则is曲线向右上方移动
c储蓄减少税收减少,则is曲线向右上方移动
d如果投资增加,则is曲线向左上方移动
e***购买增加,则is曲线向左上方移动
8.***支出的***工具包括
a税收b公债c***购买d转移支付e***投资
9.影响进口的直接因素是
a初始进口b边际进口倾向c国民收入d边际消费倾向e消费者剩余
yangshu9611年前1
yjwrfqu 共回答了14个问题 | 采纳率92.9%
3错误 4正确 5错误
DCCCA BACAC
AC ABE ABC ABC BD BC AB CDE ABC
关于IS曲线正确的是( ).A.若人们的储蓄倾向增高,则IS曲线将右移 B.若投资需求曲线向外移动,则IS曲线将左移C.
关于is曲线正确的是( ).
a.若人们的储蓄倾向增高,则is曲线将右移
b.若投资需求曲线向外移动,则is曲线将左移
c.预算平衡条件下***支出增加并不移动is曲线
d.充分就业预算盈余的减少意味着is曲线右移
为什么a不正确呢?
xlanni1年前1
狂刀2000 共回答了27个问题 | 采纳率88.9%
按照一般来理解,人们储蓄倾向增高,消费倾向降低,IS曲线应左移.
储蓄倾向升高并不一定是利率升高的结果,人们预期下一阶段物价会下跌,储蓄倾向也会升高.
详见《西方经济学(宏观部分)》人民大学出版社第四版 高鸿业 P488页IS曲线推导过程.
a的IS曲线为y=750-25r;b的IS曲线为y=750-50r HELP
a的IS曲线为y=750-25r;b的IS曲线为y=750-50r HELP
(a)的IS曲线为y=750-25r;(b)的IS曲线为y=750-50r
比较(a)和(b),说明投资对利率更敏感时,IS曲线的斜率发生什么变化
答案是((b)的投资函数中的投资对利率更敏感,表现在IS曲线上就是IS曲线斜率的绝对值变小,即IS曲线更平坦一些)
表现在IS上不是斜率绝对值变大吗50>25啊?不是更陡峭吗?
jjgsjj1年前1
beancute 共回答了16个问题 | 采纳率81.3%
IS曲线的斜率绝对值不是50和25,而是1/50和1/25.
注意:r和y分别在纵横轴,与数学习惯不同.
宏观经济学 如果IS方程为y=kA-kdr k为支出乘数 IS曲线斜率变小的原因
宏观经济学 如果IS方程为y=kA-kdr k为支出乘数 IS曲线斜率变小的原因
A k 变小,d变小
B 大 大 不要乱答,,经济学 y 是横轴,r 是纵轴
is曲线 r=A/d-y/kd 尹伯成书中选的A不得其解 ,前面符号怎么处理呀
打错了,抱歉,尹伯成书上答案是 变大 变大 书上A选项是变大 变大 我打反了,
八月的爱人1年前1
jxk9518 共回答了12个问题 | 采纳率100%
k d 变小 1/kd变大 -1/kd 变小
说明IS曲线和LM曲线如何移动才可以.
说明IS曲线和LM曲线如何移动才可以.
(1)保持产量不变而降低利率,这需要什么样的货币政策和财政政策的混合;
(2)减少产量而不改变利率,这需要什么样的政策混合
黎大书1年前1
kv8d 共回答了17个问题 | 采纳率94.1%
保持产量不变同时降低利率可以通过紧缩性财政政策和扩张性货币政策
反应在图像上即是,IS曲线向左移动,LM曲线向右移动
保持利率不变同时降低产量可以通过紧缩性财政政策和紧缩性货币政策
反应在图像上即是,IS曲线和LM曲线同时向左
算边际消费倾向如果政府购买增加了100亿 导致Is曲线移动200亿 设利率为0.2 则边际消费倾向为多少?我想知道在算M
算边际消费倾向
如果政府购买增加了100亿 导致Is曲线移动200亿 设利率为0.2 则边际消费倾向为多少?
我想知道在算MPC的时候 利率怎么用
不小心发错了 税率是0.2 不是利率……
算MPC的时候 税率怎么用
warrior451年前1
xujiewu 共回答了23个问题 | 采纳率78.3%
(令MPC=b)
Y = C+I+G 其中C 为消费 ,I 为投资,G为政府支出
= a+b(Y-tY)+I+G 其中a为必要支出,(Y-tY)为居民可支配收入,tY为税收总量
= a+bY-tbY+I+G
即 Y = a+bY-tbY+I+G ,
将含Y项左移,
可推导出 (1-b+tb)Y=a+I+G
Y=(a+I+G) / (1-b+tb)
Y=(a+I ) / (1-b+tb) + G / (1-b+bt)
在政府购买G增加时,国民收入Y的变动与(a+I ) / (1-b+tb)无关.
所以△Y=△G / (1-b+bt)
由题目可知,△Y=200亿 △G =100亿 t=0.2
将其代入上式,则可最终求的b,即MPC
注:
1,税收分为定量税和比例税.定量税一般以T表示税收总量,它是固定的,代表税收与国民收入无关.比例税一般有一个税率t,税收总量为tY,这个税收是与国民收入正相关的.
在计算个人可支配收入时要注意题目中是定量税还是比例税,一般有税率 t 时代表是比例税.
2,上述公式中的 I(投资)项一般要展开为 I = e - d r ,但是因为这道题目中和投资无关,所以不用展开.
西方经济学里is曲线和lm曲线交叉后四个象限点的意义?
西方经济学里is曲线和lm曲线交叉后四个象限点的意义?
高鸿业的原话看不懂了,打比方四个象限里最下面那个,说是i>s有超额产品需求,L>M有超额货币需求,依据是什么?有这一个,其它三个象限我可以自己推理了.《我推理的和老高不一样,用最下面象限作比方,在is曲线左边,同样利率下所取得的收入小于均衡收入,即实际收入小于均衡收入,即实际产出小于均衡产出,即产出不足,所以有超额产品需求,因收入由储蓄决定,前面实际收入小于均衡收入即实际储蓄小于均衡储蓄也即小于均衡投资故s
bupeipei1年前1
hate428 共回答了12个问题 | 采纳率100%
这个有个很简单的理解方法.例如,LM曲线下方任意位置,要想达到货币市场均衡,需要向上移动(M变大),就是说,货币供给M增加才能达到L=M,因此,LM曲线下方的点处于L》M的状态.
关于is曲线的移动is曲线的移动可分为三个因素:投资需求,政府购买的变动;储蓄意愿的变动;税收的变动.可是从公式中并不能
关于is曲线的移动
is曲线的移动可分为三个因素:投资需求,政府购买的变动;储蓄意愿的变动;税收的变动.
可是从公式中并不能分析出这些因素,是为什么?
loveshiyi1年前1
何祎彦hyylxy 共回答了14个问题 | 采纳率85.7%
当然可以从IS表达式中分析:r=[a+e+G0-bT0]/d-[1-b(1-t)]/d*Y
公式中的截距部分就说明了曲线的移动:a+e+G0-bT0
这些数量中a代表消费或储蓄意愿;e代表投资需求;G0代表政府购买;T0代表税收.
当然有些变量的变动还会导致曲线的转动,也就是斜率的变动:b、t和d.
请问 储蓄增加,IS曲线向哪边移动?移动幅度怎么求?
cecaa181年前1
free71 共回答了14个问题 | 采纳率100%
在IS曲线里,IS曲线右边的点意味着I<S,而IS曲线左边的点意味着I>S.因为这很容易理解,高利率会鼓励储蓄抑制投资.
在2部门经济里,储蓄增加,IS曲线向左移,因为投资减少使产出减少利率下降.
IS移动幅度:首先2部门经济I=S,y=c+I,c=a+ by,联立得:y=a+i/1-b,则储蓄增加△s,即等于投资减少△I,则移动幅度等于△y=△s/1-b=△I/1-b.也就是说储蓄增加移动幅度等于储蓄增量乘于乘数.
有道宏观经济学的题 政府支出增加使IS曲线右移kg•ΔG(kg是政府支出乘数),若要均衡收入变动接近于IS曲
有道宏观经济学的题
政府支出增加使IS曲线右移kg•ΔG(kg是政府支出乘数),若要均衡收入变动接近于IS曲线的移动量,则必须是( )
A.LM曲线平缓 B.IS曲线陡峭
C.LM曲线和IS曲线一样陡峭 D.LM曲线陡峭而IS曲线平缓
daisyqueen1年前1
篡霉糨糊 共回答了18个问题 | 采纳率94.4%
lm曲线平缓,政府支出造成is曲线移动的挤出效应最小,其实用排除法也能选出是A,你仔细看下后三项明显不对嘛
宏观经济学:is曲线在比例税情况下表达式是怎样的?
宏观经济学:is曲线在比例税情况下表达式是怎样的?
是y=a+i+g+btr-bty+x-m/1-b(1-t)+r吗?
假定四部门经济情况下,消费函数为c=100+0.9(1-t)y,投资函数为i=200-500r,净出口函数为nx=100-0.12y-500r,货币需求为l=0.8y-2000r,***支出g=200,税率t=0.2,名义货币供给m=800,价格水平p=1,求is曲线.
课后答案是:y=1500-2500r
我算出来不一样.
qwqw_121年前1
平静爱你 共回答了19个问题 | 采纳率84.2%
估计你被那些数字转晕了...由于是四部门经济,所以y=c+i+g+nx,把数据带进去,则(100+0.9y-0.9ty)+(200-500r)+200+(100-0.12y-500r)=y
则600-1000r+0.78y-0.9ty=y,因为t=0.2,所以600-1000r+0.6y=y,
所以y=1500-2500
在其他条件不变的情况下,政府购买增加会使IS曲线( ).
听风之临海1年前1
忌秋风 共回答了13个问题 | 采纳率92.3%
右移.可以看作相同价格下(同一水平线)产量更高(更偏向右边)
1.已知Is曲线的涵数为Y等于2800减32000r LM曲线的涵数为Y等于800加8000r 术国民收入均衡的产出水平
1.已知Is曲线的涵数为Y等于2800减32000r LM曲线的涵数为Y等于800加8000r 术国民收入均衡的产出水平和均衡利
2.设某厂商的段期成本涵数为Tc等于150加5Q减3Q的平方加Q的三次方 清分别计算Q等于20时的平均可变成本和Q等于十时的边际成本
shuimushi41年前1
liutao29 共回答了24个问题 | 采纳率95.8%
————————
1.已知Is曲线的涵数为Y等于2800减32000r LM曲线的涵数为Y等于800加8000r 术国民收入均衡的产出水平和均衡利
is:y=2800-32000r
LM:y=800+8000r
求均衡 即 IS=LM
r=0.05
产出水平 LM=1200 均衡利?没见过~
2.设某厂商的段期成本涵数为Tc等于150加5Q减3Q的平方加Q的三次方 清分别计算Q等于20时的平均可变成本和Q等于十时的边际成本
Tc=150+5Q-3Q^2+Q^3
平均可变成本= tc(Q)/Q=352.5
边际成本MC(Q)=△TC(Q)/△Q 即MC(Q)=245
完毕
什么是is曲线 几何推导
云帆儿1年前1
紫色ttzjj 共回答了12个问题 | 采纳率91.7%
这个是经济学的东西吧
是指讲产品市场上投资和储蓄相等的情况I=S
具体可以参考宏观经济学书籍 经济学中基本内容
IS曲线所表示的利率与实际国民生产总值之间的关系是一种什么关系
uiop0081年前1
orozco 共回答了23个问题 | 采纳率95.7%
反向关系,即利率高实际国民生产总值就低,在这种利率和收入组合下产品市场是均衡的,即总需求等于总供给.
当利率水平下降时,自发总支出增加/减少,IS曲线左移/右移?
当利率水平下降时,自发总支出增加/减少,IS曲线左移/右移?
5、当利率水平下降时()
a.自发总支出减少,IS曲线向右方移动
b.自发总支出减少,IS曲线向左方移动
c.自发总支出增加,IS曲线向左方移动
d.自发总支出增加,IS曲线向右方移动
我又回到人间1年前1
做人与做文 共回答了13个问题 | 采纳率84.6%
利率下降,投资支出会增加,因此自发总支出增加,IS曲线向右移动.因此选D
为什么西方经济学中IS曲线的右上方会有投资小于储蓄
qiuxuhappy1年前1
joejoejuju 共回答了22个问题 | 采纳率95.5%
你可以画一条垂线,随便经过IS曲线上面的任何一点,假设该点为A点,对应了 均衡收入y0和均衡利率i0,该组合表示在收入为y0下,只有利率为i0才可以使得商品市场均衡,现在假设有一点在IS曲线右上方,所以我们干脆就找一个在A点上方的点B(就是在我上来画的垂线上面),可知相对比均衡点A,B在同一收入下却有着更高的利率,由于储蓄是收入的正比函数,可知A,B两点储蓄是一样的,但是由于投资是利率的反比函数,可知B点所对应的投资小于A点对应的均衡投资,由于在A点 ,储蓄等于投资,所以在B点,投资就小于储蓄了

大家在问